Bilinear Transformation problem

  • Thread starter Thread starter RJLiberator
  • Start date Start date
  • Tags Tags
    Transformation
Click For Summary
The discussion revolves around finding the bilinear transformation that maps specific points in the complex plane. The transformation is expressed as w=(az+b)/(cz+d), leading to the conclusion that d must be 0 since 0 maps to infinity. The limit as z approaches infinity indicates that a must also be 0, simplifying the equation to b/cz. The participants clarify that the specific values of b and c are not crucial, as they can be scaled without affecting the transformation, ultimately confirming that the solution is -1/z. The focus is on understanding the nature of bilinear transformations and their properties rather than proving specific constants.
RJLiberator
Gold Member
Messages
1,094
Reaction score
63

Homework Statement


Find the bilinear transformation that maps the points z1=infinity, z2=i, z3=0 to the points w1=0, w2=i, w3=infinity.

Homework Equations


w=(az+b)/(cz+d)

The answer is -1/z

The Attempt at a Solution



We have:
infinity --> 0
i --> i
0 --> infinity

Since 0 goes to infinity, it means the denominator "is 0", so therefore d must be 0 in our relevant equation right off the bat.

If we take the limit as z--> infinity, we are supposed to get w=0, so 0=a/c in the limit and we see that a=0.

So now we are left with b/cz
and i-->

i=b/(ci) and we see that -c=b

But how do I prove here that c=1, and b=-1 instead of something else?

thanks.
 
Physics news on Phys.org
RJLiberator said:
But how do I prove here that c=1, and b=-1 instead of something else?
This is irrelevant. If you take c = 2 and b = -2, you end up with the same transformation (i.e., the constants are only well defined up to an overall multiplicative factor).
 
Hm. Excellent, so what you are saying is it does not need to be proved since it's irrelevant.
It may as well be c=100, and b=-100 and then w'ed have
-100/(100z) which reduces to -1/z

Thanks.
 
Question: A clock's minute hand has length 4 and its hour hand has length 3. What is the distance between the tips at the moment when it is increasing most rapidly?(Putnam Exam Question) Answer: Making assumption that both the hands moves at constant angular velocities, the answer is ## \sqrt{7} .## But don't you think this assumption is somewhat doubtful and wrong?

Similar threads

  • · Replies 2 ·
Replies
2
Views
1K
  • · Replies 1 ·
Replies
1
Views
1K
Replies
4
Views
2K
Replies
2
Views
2K
  • · Replies 2 ·
Replies
2
Views
2K
  • · Replies 2 ·
Replies
2
Views
2K
  • · Replies 7 ·
Replies
7
Views
2K
  • · Replies 2 ·
Replies
2
Views
2K
Replies
1
Views
2K
  • · Replies 17 ·
Replies
17
Views
2K